hook-length formula: “Fibonaccized”hook-length formula: “Fibonaccized”: Part IINekrasov-Okounkov hook length formulaInequality for hook numbers in Young diagramspartitions into odd parts vs hooks and symplectic contentsan identity for a sum over partitionsLemmas involving two partitions of integershooks and contents: Part IA discrete operator begets even/odd polynomialsPartitions, $q$-polynomials and generating functionsPartitions and $q$-integersA link between hooks, contents and parts of a partition

hook-length formula: “Fibonaccized”


hook-length formula: “Fibonaccized”: Part IINekrasov-Okounkov hook length formulaInequality for hook numbers in Young diagramspartitions into odd parts vs hooks and symplectic contentsan identity for a sum over partitionsLemmas involving two partitions of integershooks and contents: Part IA discrete operator begets even/odd polynomialsPartitions, $q$-polynomials and generating functionsPartitions and $q$-integersA link between hooks, contents and parts of a partition













16












$begingroup$


Consider the Young diagram of a partition $lambda = (lambda_1,ldots,lambda_k)$. For a square $(i,j) in lambda$, define the hook numbers $h_(i,j) = lambda_i + lambda_j' -i - j +1$ where $lambda'$ is the conjugate of $lambda$.



The hook-length formula shows, in particular, that if $lambdavdash n$ then
$$text$n!prod_square,in,lambdafrac1h_square$ qquad textis an integer.$$
Recall the Fibonacci numbers $F(0)=0, , F(1)=1$ with $F(n)=F(n-1)+F(n-2)$. Define $[0]!_F=1$ and $[n]!_F=F(1)cdot F(2)cdots F(n)$ for $ngeq1$.




QUESTION. Is it true that
$$text$[n]!_Fprod_square,in,lambdafrac1F(h_square)$ qquad textis an integer?$$











share|cite|improve this question











$endgroup$







  • 5




    $begingroup$
    More generally, this interesting question can be asked of any strong divisibility sequence instead of the Fibonacci sequence. But let's perhaps not abuse the notation $Fleft(nright)!$ for something that's not the factorial of $Fleft(nright)$.
    $endgroup$
    – darij grinberg
    2 days ago







  • 1




    $begingroup$
    Maybe call it $F!(n)$ instead of $F(n)!$. How far has this been checked?
    $endgroup$
    – Noam D. Elkies
    2 days ago







  • 9




    $begingroup$
    Maybe this expression can be obtained by a clever substitution of the $q$-hook length formula?
    $endgroup$
    – Sam Hopkins
    2 days ago






  • 1




    $begingroup$
    @darijgrinberg what is a strong divisibility sequence? Product of any $k$ consecutive guys is divisibly by the product of first $k$ guys?
    $endgroup$
    – Fedor Petrov
    yesterday






  • 5




    $begingroup$
    For searching purposes: the product of consecutive Fibonacci numbers is sometimes referred to as a fibonorial.
    $endgroup$
    – J. M. is not a mathematician
    yesterday















16












$begingroup$


Consider the Young diagram of a partition $lambda = (lambda_1,ldots,lambda_k)$. For a square $(i,j) in lambda$, define the hook numbers $h_(i,j) = lambda_i + lambda_j' -i - j +1$ where $lambda'$ is the conjugate of $lambda$.



The hook-length formula shows, in particular, that if $lambdavdash n$ then
$$text$n!prod_square,in,lambdafrac1h_square$ qquad textis an integer.$$
Recall the Fibonacci numbers $F(0)=0, , F(1)=1$ with $F(n)=F(n-1)+F(n-2)$. Define $[0]!_F=1$ and $[n]!_F=F(1)cdot F(2)cdots F(n)$ for $ngeq1$.




QUESTION. Is it true that
$$text$[n]!_Fprod_square,in,lambdafrac1F(h_square)$ qquad textis an integer?$$











share|cite|improve this question











$endgroup$







  • 5




    $begingroup$
    More generally, this interesting question can be asked of any strong divisibility sequence instead of the Fibonacci sequence. But let's perhaps not abuse the notation $Fleft(nright)!$ for something that's not the factorial of $Fleft(nright)$.
    $endgroup$
    – darij grinberg
    2 days ago







  • 1




    $begingroup$
    Maybe call it $F!(n)$ instead of $F(n)!$. How far has this been checked?
    $endgroup$
    – Noam D. Elkies
    2 days ago







  • 9




    $begingroup$
    Maybe this expression can be obtained by a clever substitution of the $q$-hook length formula?
    $endgroup$
    – Sam Hopkins
    2 days ago






  • 1




    $begingroup$
    @darijgrinberg what is a strong divisibility sequence? Product of any $k$ consecutive guys is divisibly by the product of first $k$ guys?
    $endgroup$
    – Fedor Petrov
    yesterday






  • 5




    $begingroup$
    For searching purposes: the product of consecutive Fibonacci numbers is sometimes referred to as a fibonorial.
    $endgroup$
    – J. M. is not a mathematician
    yesterday













16












16








16


6



$begingroup$


Consider the Young diagram of a partition $lambda = (lambda_1,ldots,lambda_k)$. For a square $(i,j) in lambda$, define the hook numbers $h_(i,j) = lambda_i + lambda_j' -i - j +1$ where $lambda'$ is the conjugate of $lambda$.



The hook-length formula shows, in particular, that if $lambdavdash n$ then
$$text$n!prod_square,in,lambdafrac1h_square$ qquad textis an integer.$$
Recall the Fibonacci numbers $F(0)=0, , F(1)=1$ with $F(n)=F(n-1)+F(n-2)$. Define $[0]!_F=1$ and $[n]!_F=F(1)cdot F(2)cdots F(n)$ for $ngeq1$.




QUESTION. Is it true that
$$text$[n]!_Fprod_square,in,lambdafrac1F(h_square)$ qquad textis an integer?$$











share|cite|improve this question











$endgroup$




Consider the Young diagram of a partition $lambda = (lambda_1,ldots,lambda_k)$. For a square $(i,j) in lambda$, define the hook numbers $h_(i,j) = lambda_i + lambda_j' -i - j +1$ where $lambda'$ is the conjugate of $lambda$.



The hook-length formula shows, in particular, that if $lambdavdash n$ then
$$text$n!prod_square,in,lambdafrac1h_square$ qquad textis an integer.$$
Recall the Fibonacci numbers $F(0)=0, , F(1)=1$ with $F(n)=F(n-1)+F(n-2)$. Define $[0]!_F=1$ and $[n]!_F=F(1)cdot F(2)cdots F(n)$ for $ngeq1$.




QUESTION. Is it true that
$$text$[n]!_Fprod_square,in,lambdafrac1F(h_square)$ qquad textis an integer?$$








nt.number-theory co.combinatorics partitions algebraic-combinatorics






share|cite|improve this question















share|cite|improve this question













share|cite|improve this question




share|cite|improve this question








edited 2 days ago







T. Amdeberhan

















asked 2 days ago









T. AmdeberhanT. Amdeberhan

18.1k229132




18.1k229132







  • 5




    $begingroup$
    More generally, this interesting question can be asked of any strong divisibility sequence instead of the Fibonacci sequence. But let's perhaps not abuse the notation $Fleft(nright)!$ for something that's not the factorial of $Fleft(nright)$.
    $endgroup$
    – darij grinberg
    2 days ago







  • 1




    $begingroup$
    Maybe call it $F!(n)$ instead of $F(n)!$. How far has this been checked?
    $endgroup$
    – Noam D. Elkies
    2 days ago







  • 9




    $begingroup$
    Maybe this expression can be obtained by a clever substitution of the $q$-hook length formula?
    $endgroup$
    – Sam Hopkins
    2 days ago






  • 1




    $begingroup$
    @darijgrinberg what is a strong divisibility sequence? Product of any $k$ consecutive guys is divisibly by the product of first $k$ guys?
    $endgroup$
    – Fedor Petrov
    yesterday






  • 5




    $begingroup$
    For searching purposes: the product of consecutive Fibonacci numbers is sometimes referred to as a fibonorial.
    $endgroup$
    – J. M. is not a mathematician
    yesterday












  • 5




    $begingroup$
    More generally, this interesting question can be asked of any strong divisibility sequence instead of the Fibonacci sequence. But let's perhaps not abuse the notation $Fleft(nright)!$ for something that's not the factorial of $Fleft(nright)$.
    $endgroup$
    – darij grinberg
    2 days ago







  • 1




    $begingroup$
    Maybe call it $F!(n)$ instead of $F(n)!$. How far has this been checked?
    $endgroup$
    – Noam D. Elkies
    2 days ago







  • 9




    $begingroup$
    Maybe this expression can be obtained by a clever substitution of the $q$-hook length formula?
    $endgroup$
    – Sam Hopkins
    2 days ago






  • 1




    $begingroup$
    @darijgrinberg what is a strong divisibility sequence? Product of any $k$ consecutive guys is divisibly by the product of first $k$ guys?
    $endgroup$
    – Fedor Petrov
    yesterday






  • 5




    $begingroup$
    For searching purposes: the product of consecutive Fibonacci numbers is sometimes referred to as a fibonorial.
    $endgroup$
    – J. M. is not a mathematician
    yesterday







5




5




$begingroup$
More generally, this interesting question can be asked of any strong divisibility sequence instead of the Fibonacci sequence. But let's perhaps not abuse the notation $Fleft(nright)!$ for something that's not the factorial of $Fleft(nright)$.
$endgroup$
– darij grinberg
2 days ago





$begingroup$
More generally, this interesting question can be asked of any strong divisibility sequence instead of the Fibonacci sequence. But let's perhaps not abuse the notation $Fleft(nright)!$ for something that's not the factorial of $Fleft(nright)$.
$endgroup$
– darij grinberg
2 days ago





1




1




$begingroup$
Maybe call it $F!(n)$ instead of $F(n)!$. How far has this been checked?
$endgroup$
– Noam D. Elkies
2 days ago





$begingroup$
Maybe call it $F!(n)$ instead of $F(n)!$. How far has this been checked?
$endgroup$
– Noam D. Elkies
2 days ago





9




9




$begingroup$
Maybe this expression can be obtained by a clever substitution of the $q$-hook length formula?
$endgroup$
– Sam Hopkins
2 days ago




$begingroup$
Maybe this expression can be obtained by a clever substitution of the $q$-hook length formula?
$endgroup$
– Sam Hopkins
2 days ago




1




1




$begingroup$
@darijgrinberg what is a strong divisibility sequence? Product of any $k$ consecutive guys is divisibly by the product of first $k$ guys?
$endgroup$
– Fedor Petrov
yesterday




$begingroup$
@darijgrinberg what is a strong divisibility sequence? Product of any $k$ consecutive guys is divisibly by the product of first $k$ guys?
$endgroup$
– Fedor Petrov
yesterday




5




5




$begingroup$
For searching purposes: the product of consecutive Fibonacci numbers is sometimes referred to as a fibonorial.
$endgroup$
– J. M. is not a mathematician
yesterday




$begingroup$
For searching purposes: the product of consecutive Fibonacci numbers is sometimes referred to as a fibonorial.
$endgroup$
– J. M. is not a mathematician
yesterday










2 Answers
2






active

oldest

votes


















10












$begingroup$

Sam is correct of course about $q$-hook formula. Below is a short self-contained proof not relying on such advanced combinatorics.



Denote $h_1>ldots>h_k$ the set of hook lengths of the first column of diagram $lambda$. Then the multiset of hooks is $cup_i=1^k 1,2,ldots,h_isetminus h_i-h_j:i<j$ and $n=sum_i h_i-frack(k-1)2$.



Recall that $F(m)=P_m(alpha,beta)=prod_dPhi_d(alpha,beta)=prod_d (Phi_d(alpha,beta))^eta_d(m)$, where



$alpha,beta=(1pm sqrt5)/2$;



$P_n(x,y)=x^n-1+x^n-2y+ldots+y^n-1$;



$Phi_d$ are homogeneous cyclotomic polynomials;



$eta_d(m)=chi_mathbbZ(m/d)$ (i.e., it equals to 1 if $d$ divides $m$, and to 0 otherwise).



Therefore it suffices to prove that for any fixed $d>1$ we have
$$
sum_m=1^n eta_d(m)+sum_i<jeta_d(h_i-h_j)-sum_i=1^ksum_j=1^h_ieta_d(j)geqslant 0.quad (ast)
$$

$(ast)$ rewrites as
$$
[n/d]+|i<j:h_iequiv h_j pmod d|-sum_i=1^k [h_i/d]geqslant 0.quad (bullet)
$$

LHS of $(bullet)$ does not change if we reduce all $h_i$'s modulo $d$ (and accordingly change $n=sum_i h_i-frack(k-1)2$, of course), so we may suppose that $0leqslant h_ileqslant d-1$ for all $i$. For $a=0,1,dots, d-1$ denote $t_a=|i:h_i=a|$. Then $(bullet)$ rewrites as
$$
left[frac-binomsum_i=0^d-1 t_i2+sum_i=0^d-1 it_idright]+
sum_i=0^d-1 binomt_i2geqslant 0. quad (star)
$$



It remains to observe that LHS of $(star)$ equals to
$$
left[frac1dsum_i<jbinomt_i-t_j2 right].
$$






share|cite|improve this answer











$endgroup$












  • $begingroup$
    Nice. So what does this integer count?
    $endgroup$
    – Brian Hopkins
    yesterday










  • $begingroup$
    Yes, that was my plan to ask next.
    $endgroup$
    – T. Amdeberhan
    yesterday











  • $begingroup$
    @Fedor: what are $alpha$ and $beta$? What's the connection between $P_m$ and $eta_d$, etc?
    $endgroup$
    – T. Amdeberhan
    12 hours ago










  • $begingroup$
    @T.Amdeberhan sorry, forgot to copy the notations from my previous Fibonacci answer. Hope now it is clear.
    $endgroup$
    – Fedor Petrov
    11 hours ago


















2












$begingroup$

This is a less elementary but maybe more conceptual proof, also giving some combinatorial meaning:
Use the formulas
$F(n) = fracvarphi^n -psi^nsqrt5$, $varphi =frac1+sqrt52, psi = frac1-sqrt52$. Let $q=fracpsivarphi = fracsqrt5-32$, so that
$F(n) = fracvarphi^nsqrt5 (1-q^n)$



Then the Fibonacci hook-length formula becomes:
DeclareMathOperatorlalambda



beginalign*
f^lambda_F:= frac[n]!_Fprod_uin lambdaF(h(u)) = frac varphi^ binomn+12 [n]!_q varphi^sum_u in lambda h(u) prod_u in lambda (1-q^h(u))
endalign*

So we have an ordinary $q$-analogue of the hook-length formula. Note that
$$sum_u in lambda h(u) = sum_i binomlambda_i2 + binomlambda'_j2 + |lambda| = b(lambda) +b(lambda') +n$$
Using the $q-$analogue hook-length formula via major index (EC2, Chapter 21) we have



beginalign*
f^lambda_F = varphi^ binomn2 -b(lambda)-b(lambda') q^-b(lambda) sum_Tin SYT(lambda) q^maj(T) = (-q)^frac12( -binomn2 +b(lambda') -b(lambda))sum_T q^maj(T)
endalign*



Now, it is clear from the q-HLF formula that $q^maj(T)$ is a symmetric polynomial, with lowest degree term $b(lambda)$ and maximal degree $b(lambda) + binomn+12 - n -b(lambda) -b(lambda') =binomn2 - b(lambda')$ so the median degree term is
$$M=frac12 left(b(lambda) +binomn2 - b(lambda')right)$$
which cancels with the factor of $q$ in $f^lambda_F$, so the resulting polynomial is of the form
beginalign*
f^lambda_F = (-1)^M sum_T: maj(T) leq M (q^M-maj(T) + q^maj(T)-M) \
= (-1)^M sum_T (-1)^M-maj(T)( varphi^2(M-maj(T)) + psi^2(M-maj(T)) =
sum_T (-1)^maj(T) L(2(M-maj(T)))
endalign*

where $L$ are the Lucas numbers.



**byproduct of collaborations with A. Morales and I. Pak.






share|cite|improve this answer









$endgroup$













    Your Answer





    StackExchange.ifUsing("editor", function ()
    return StackExchange.using("mathjaxEditing", function ()
    StackExchange.MarkdownEditor.creationCallbacks.add(function (editor, postfix)
    StackExchange.mathjaxEditing.prepareWmdForMathJax(editor, postfix, [["$", "$"], ["\\(","\\)"]]);
    );
    );
    , "mathjax-editing");

    StackExchange.ready(function()
    var channelOptions =
    tags: "".split(" "),
    id: "504"
    ;
    initTagRenderer("".split(" "), "".split(" "), channelOptions);

    StackExchange.using("externalEditor", function()
    // Have to fire editor after snippets, if snippets enabled
    if (StackExchange.settings.snippets.snippetsEnabled)
    StackExchange.using("snippets", function()
    createEditor();
    );

    else
    createEditor();

    );

    function createEditor()
    StackExchange.prepareEditor(
    heartbeatType: 'answer',
    autoActivateHeartbeat: false,
    convertImagesToLinks: true,
    noModals: true,
    showLowRepImageUploadWarning: true,
    reputationToPostImages: 10,
    bindNavPrevention: true,
    postfix: "",
    imageUploader:
    brandingHtml: "Powered by u003ca class="icon-imgur-white" href="https://imgur.com/"u003eu003c/au003e",
    contentPolicyHtml: "User contributions licensed under u003ca href="https://creativecommons.org/licenses/by-sa/3.0/"u003ecc by-sa 3.0 with attribution requiredu003c/au003e u003ca href="https://stackoverflow.com/legal/content-policy"u003e(content policy)u003c/au003e",
    allowUrls: true
    ,
    noCode: true, onDemand: true,
    discardSelector: ".discard-answer"
    ,immediatelyShowMarkdownHelp:true
    );



    );













    draft saved

    draft discarded


















    StackExchange.ready(
    function ()
    StackExchange.openid.initPostLogin('.new-post-login', 'https%3a%2f%2fmathoverflow.net%2fquestions%2f326860%2fhook-length-formula-fibonaccized%23new-answer', 'question_page');

    );

    Post as a guest















    Required, but never shown

























    2 Answers
    2






    active

    oldest

    votes








    2 Answers
    2






    active

    oldest

    votes









    active

    oldest

    votes






    active

    oldest

    votes









    10












    $begingroup$

    Sam is correct of course about $q$-hook formula. Below is a short self-contained proof not relying on such advanced combinatorics.



    Denote $h_1>ldots>h_k$ the set of hook lengths of the first column of diagram $lambda$. Then the multiset of hooks is $cup_i=1^k 1,2,ldots,h_isetminus h_i-h_j:i<j$ and $n=sum_i h_i-frack(k-1)2$.



    Recall that $F(m)=P_m(alpha,beta)=prod_dPhi_d(alpha,beta)=prod_d (Phi_d(alpha,beta))^eta_d(m)$, where



    $alpha,beta=(1pm sqrt5)/2$;



    $P_n(x,y)=x^n-1+x^n-2y+ldots+y^n-1$;



    $Phi_d$ are homogeneous cyclotomic polynomials;



    $eta_d(m)=chi_mathbbZ(m/d)$ (i.e., it equals to 1 if $d$ divides $m$, and to 0 otherwise).



    Therefore it suffices to prove that for any fixed $d>1$ we have
    $$
    sum_m=1^n eta_d(m)+sum_i<jeta_d(h_i-h_j)-sum_i=1^ksum_j=1^h_ieta_d(j)geqslant 0.quad (ast)
    $$

    $(ast)$ rewrites as
    $$
    [n/d]+|i<j:h_iequiv h_j pmod d|-sum_i=1^k [h_i/d]geqslant 0.quad (bullet)
    $$

    LHS of $(bullet)$ does not change if we reduce all $h_i$'s modulo $d$ (and accordingly change $n=sum_i h_i-frack(k-1)2$, of course), so we may suppose that $0leqslant h_ileqslant d-1$ for all $i$. For $a=0,1,dots, d-1$ denote $t_a=|i:h_i=a|$. Then $(bullet)$ rewrites as
    $$
    left[frac-binomsum_i=0^d-1 t_i2+sum_i=0^d-1 it_idright]+
    sum_i=0^d-1 binomt_i2geqslant 0. quad (star)
    $$



    It remains to observe that LHS of $(star)$ equals to
    $$
    left[frac1dsum_i<jbinomt_i-t_j2 right].
    $$






    share|cite|improve this answer











    $endgroup$












    • $begingroup$
      Nice. So what does this integer count?
      $endgroup$
      – Brian Hopkins
      yesterday










    • $begingroup$
      Yes, that was my plan to ask next.
      $endgroup$
      – T. Amdeberhan
      yesterday











    • $begingroup$
      @Fedor: what are $alpha$ and $beta$? What's the connection between $P_m$ and $eta_d$, etc?
      $endgroup$
      – T. Amdeberhan
      12 hours ago










    • $begingroup$
      @T.Amdeberhan sorry, forgot to copy the notations from my previous Fibonacci answer. Hope now it is clear.
      $endgroup$
      – Fedor Petrov
      11 hours ago















    10












    $begingroup$

    Sam is correct of course about $q$-hook formula. Below is a short self-contained proof not relying on such advanced combinatorics.



    Denote $h_1>ldots>h_k$ the set of hook lengths of the first column of diagram $lambda$. Then the multiset of hooks is $cup_i=1^k 1,2,ldots,h_isetminus h_i-h_j:i<j$ and $n=sum_i h_i-frack(k-1)2$.



    Recall that $F(m)=P_m(alpha,beta)=prod_dPhi_d(alpha,beta)=prod_d (Phi_d(alpha,beta))^eta_d(m)$, where



    $alpha,beta=(1pm sqrt5)/2$;



    $P_n(x,y)=x^n-1+x^n-2y+ldots+y^n-1$;



    $Phi_d$ are homogeneous cyclotomic polynomials;



    $eta_d(m)=chi_mathbbZ(m/d)$ (i.e., it equals to 1 if $d$ divides $m$, and to 0 otherwise).



    Therefore it suffices to prove that for any fixed $d>1$ we have
    $$
    sum_m=1^n eta_d(m)+sum_i<jeta_d(h_i-h_j)-sum_i=1^ksum_j=1^h_ieta_d(j)geqslant 0.quad (ast)
    $$

    $(ast)$ rewrites as
    $$
    [n/d]+|i<j:h_iequiv h_j pmod d|-sum_i=1^k [h_i/d]geqslant 0.quad (bullet)
    $$

    LHS of $(bullet)$ does not change if we reduce all $h_i$'s modulo $d$ (and accordingly change $n=sum_i h_i-frack(k-1)2$, of course), so we may suppose that $0leqslant h_ileqslant d-1$ for all $i$. For $a=0,1,dots, d-1$ denote $t_a=|i:h_i=a|$. Then $(bullet)$ rewrites as
    $$
    left[frac-binomsum_i=0^d-1 t_i2+sum_i=0^d-1 it_idright]+
    sum_i=0^d-1 binomt_i2geqslant 0. quad (star)
    $$



    It remains to observe that LHS of $(star)$ equals to
    $$
    left[frac1dsum_i<jbinomt_i-t_j2 right].
    $$






    share|cite|improve this answer











    $endgroup$












    • $begingroup$
      Nice. So what does this integer count?
      $endgroup$
      – Brian Hopkins
      yesterday










    • $begingroup$
      Yes, that was my plan to ask next.
      $endgroup$
      – T. Amdeberhan
      yesterday











    • $begingroup$
      @Fedor: what are $alpha$ and $beta$? What's the connection between $P_m$ and $eta_d$, etc?
      $endgroup$
      – T. Amdeberhan
      12 hours ago










    • $begingroup$
      @T.Amdeberhan sorry, forgot to copy the notations from my previous Fibonacci answer. Hope now it is clear.
      $endgroup$
      – Fedor Petrov
      11 hours ago













    10












    10








    10





    $begingroup$

    Sam is correct of course about $q$-hook formula. Below is a short self-contained proof not relying on such advanced combinatorics.



    Denote $h_1>ldots>h_k$ the set of hook lengths of the first column of diagram $lambda$. Then the multiset of hooks is $cup_i=1^k 1,2,ldots,h_isetminus h_i-h_j:i<j$ and $n=sum_i h_i-frack(k-1)2$.



    Recall that $F(m)=P_m(alpha,beta)=prod_dPhi_d(alpha,beta)=prod_d (Phi_d(alpha,beta))^eta_d(m)$, where



    $alpha,beta=(1pm sqrt5)/2$;



    $P_n(x,y)=x^n-1+x^n-2y+ldots+y^n-1$;



    $Phi_d$ are homogeneous cyclotomic polynomials;



    $eta_d(m)=chi_mathbbZ(m/d)$ (i.e., it equals to 1 if $d$ divides $m$, and to 0 otherwise).



    Therefore it suffices to prove that for any fixed $d>1$ we have
    $$
    sum_m=1^n eta_d(m)+sum_i<jeta_d(h_i-h_j)-sum_i=1^ksum_j=1^h_ieta_d(j)geqslant 0.quad (ast)
    $$

    $(ast)$ rewrites as
    $$
    [n/d]+|i<j:h_iequiv h_j pmod d|-sum_i=1^k [h_i/d]geqslant 0.quad (bullet)
    $$

    LHS of $(bullet)$ does not change if we reduce all $h_i$'s modulo $d$ (and accordingly change $n=sum_i h_i-frack(k-1)2$, of course), so we may suppose that $0leqslant h_ileqslant d-1$ for all $i$. For $a=0,1,dots, d-1$ denote $t_a=|i:h_i=a|$. Then $(bullet)$ rewrites as
    $$
    left[frac-binomsum_i=0^d-1 t_i2+sum_i=0^d-1 it_idright]+
    sum_i=0^d-1 binomt_i2geqslant 0. quad (star)
    $$



    It remains to observe that LHS of $(star)$ equals to
    $$
    left[frac1dsum_i<jbinomt_i-t_j2 right].
    $$






    share|cite|improve this answer











    $endgroup$



    Sam is correct of course about $q$-hook formula. Below is a short self-contained proof not relying on such advanced combinatorics.



    Denote $h_1>ldots>h_k$ the set of hook lengths of the first column of diagram $lambda$. Then the multiset of hooks is $cup_i=1^k 1,2,ldots,h_isetminus h_i-h_j:i<j$ and $n=sum_i h_i-frack(k-1)2$.



    Recall that $F(m)=P_m(alpha,beta)=prod_dPhi_d(alpha,beta)=prod_d (Phi_d(alpha,beta))^eta_d(m)$, where



    $alpha,beta=(1pm sqrt5)/2$;



    $P_n(x,y)=x^n-1+x^n-2y+ldots+y^n-1$;



    $Phi_d$ are homogeneous cyclotomic polynomials;



    $eta_d(m)=chi_mathbbZ(m/d)$ (i.e., it equals to 1 if $d$ divides $m$, and to 0 otherwise).



    Therefore it suffices to prove that for any fixed $d>1$ we have
    $$
    sum_m=1^n eta_d(m)+sum_i<jeta_d(h_i-h_j)-sum_i=1^ksum_j=1^h_ieta_d(j)geqslant 0.quad (ast)
    $$

    $(ast)$ rewrites as
    $$
    [n/d]+|i<j:h_iequiv h_j pmod d|-sum_i=1^k [h_i/d]geqslant 0.quad (bullet)
    $$

    LHS of $(bullet)$ does not change if we reduce all $h_i$'s modulo $d$ (and accordingly change $n=sum_i h_i-frack(k-1)2$, of course), so we may suppose that $0leqslant h_ileqslant d-1$ for all $i$. For $a=0,1,dots, d-1$ denote $t_a=|i:h_i=a|$. Then $(bullet)$ rewrites as
    $$
    left[frac-binomsum_i=0^d-1 t_i2+sum_i=0^d-1 it_idright]+
    sum_i=0^d-1 binomt_i2geqslant 0. quad (star)
    $$



    It remains to observe that LHS of $(star)$ equals to
    $$
    left[frac1dsum_i<jbinomt_i-t_j2 right].
    $$







    share|cite|improve this answer














    share|cite|improve this answer



    share|cite|improve this answer








    edited 11 hours ago

























    answered yesterday









    Fedor PetrovFedor Petrov

    51.5k6120237




    51.5k6120237











    • $begingroup$
      Nice. So what does this integer count?
      $endgroup$
      – Brian Hopkins
      yesterday










    • $begingroup$
      Yes, that was my plan to ask next.
      $endgroup$
      – T. Amdeberhan
      yesterday











    • $begingroup$
      @Fedor: what are $alpha$ and $beta$? What's the connection between $P_m$ and $eta_d$, etc?
      $endgroup$
      – T. Amdeberhan
      12 hours ago










    • $begingroup$
      @T.Amdeberhan sorry, forgot to copy the notations from my previous Fibonacci answer. Hope now it is clear.
      $endgroup$
      – Fedor Petrov
      11 hours ago
















    • $begingroup$
      Nice. So what does this integer count?
      $endgroup$
      – Brian Hopkins
      yesterday










    • $begingroup$
      Yes, that was my plan to ask next.
      $endgroup$
      – T. Amdeberhan
      yesterday











    • $begingroup$
      @Fedor: what are $alpha$ and $beta$? What's the connection between $P_m$ and $eta_d$, etc?
      $endgroup$
      – T. Amdeberhan
      12 hours ago










    • $begingroup$
      @T.Amdeberhan sorry, forgot to copy the notations from my previous Fibonacci answer. Hope now it is clear.
      $endgroup$
      – Fedor Petrov
      11 hours ago















    $begingroup$
    Nice. So what does this integer count?
    $endgroup$
    – Brian Hopkins
    yesterday




    $begingroup$
    Nice. So what does this integer count?
    $endgroup$
    – Brian Hopkins
    yesterday












    $begingroup$
    Yes, that was my plan to ask next.
    $endgroup$
    – T. Amdeberhan
    yesterday





    $begingroup$
    Yes, that was my plan to ask next.
    $endgroup$
    – T. Amdeberhan
    yesterday













    $begingroup$
    @Fedor: what are $alpha$ and $beta$? What's the connection between $P_m$ and $eta_d$, etc?
    $endgroup$
    – T. Amdeberhan
    12 hours ago




    $begingroup$
    @Fedor: what are $alpha$ and $beta$? What's the connection between $P_m$ and $eta_d$, etc?
    $endgroup$
    – T. Amdeberhan
    12 hours ago












    $begingroup$
    @T.Amdeberhan sorry, forgot to copy the notations from my previous Fibonacci answer. Hope now it is clear.
    $endgroup$
    – Fedor Petrov
    11 hours ago




    $begingroup$
    @T.Amdeberhan sorry, forgot to copy the notations from my previous Fibonacci answer. Hope now it is clear.
    $endgroup$
    – Fedor Petrov
    11 hours ago











    2












    $begingroup$

    This is a less elementary but maybe more conceptual proof, also giving some combinatorial meaning:
    Use the formulas
    $F(n) = fracvarphi^n -psi^nsqrt5$, $varphi =frac1+sqrt52, psi = frac1-sqrt52$. Let $q=fracpsivarphi = fracsqrt5-32$, so that
    $F(n) = fracvarphi^nsqrt5 (1-q^n)$



    Then the Fibonacci hook-length formula becomes:
    DeclareMathOperatorlalambda



    beginalign*
    f^lambda_F:= frac[n]!_Fprod_uin lambdaF(h(u)) = frac varphi^ binomn+12 [n]!_q varphi^sum_u in lambda h(u) prod_u in lambda (1-q^h(u))
    endalign*

    So we have an ordinary $q$-analogue of the hook-length formula. Note that
    $$sum_u in lambda h(u) = sum_i binomlambda_i2 + binomlambda'_j2 + |lambda| = b(lambda) +b(lambda') +n$$
    Using the $q-$analogue hook-length formula via major index (EC2, Chapter 21) we have



    beginalign*
    f^lambda_F = varphi^ binomn2 -b(lambda)-b(lambda') q^-b(lambda) sum_Tin SYT(lambda) q^maj(T) = (-q)^frac12( -binomn2 +b(lambda') -b(lambda))sum_T q^maj(T)
    endalign*



    Now, it is clear from the q-HLF formula that $q^maj(T)$ is a symmetric polynomial, with lowest degree term $b(lambda)$ and maximal degree $b(lambda) + binomn+12 - n -b(lambda) -b(lambda') =binomn2 - b(lambda')$ so the median degree term is
    $$M=frac12 left(b(lambda) +binomn2 - b(lambda')right)$$
    which cancels with the factor of $q$ in $f^lambda_F$, so the resulting polynomial is of the form
    beginalign*
    f^lambda_F = (-1)^M sum_T: maj(T) leq M (q^M-maj(T) + q^maj(T)-M) \
    = (-1)^M sum_T (-1)^M-maj(T)( varphi^2(M-maj(T)) + psi^2(M-maj(T)) =
    sum_T (-1)^maj(T) L(2(M-maj(T)))
    endalign*

    where $L$ are the Lucas numbers.



    **byproduct of collaborations with A. Morales and I. Pak.






    share|cite|improve this answer









    $endgroup$

















      2












      $begingroup$

      This is a less elementary but maybe more conceptual proof, also giving some combinatorial meaning:
      Use the formulas
      $F(n) = fracvarphi^n -psi^nsqrt5$, $varphi =frac1+sqrt52, psi = frac1-sqrt52$. Let $q=fracpsivarphi = fracsqrt5-32$, so that
      $F(n) = fracvarphi^nsqrt5 (1-q^n)$



      Then the Fibonacci hook-length formula becomes:
      DeclareMathOperatorlalambda



      beginalign*
      f^lambda_F:= frac[n]!_Fprod_uin lambdaF(h(u)) = frac varphi^ binomn+12 [n]!_q varphi^sum_u in lambda h(u) prod_u in lambda (1-q^h(u))
      endalign*

      So we have an ordinary $q$-analogue of the hook-length formula. Note that
      $$sum_u in lambda h(u) = sum_i binomlambda_i2 + binomlambda'_j2 + |lambda| = b(lambda) +b(lambda') +n$$
      Using the $q-$analogue hook-length formula via major index (EC2, Chapter 21) we have



      beginalign*
      f^lambda_F = varphi^ binomn2 -b(lambda)-b(lambda') q^-b(lambda) sum_Tin SYT(lambda) q^maj(T) = (-q)^frac12( -binomn2 +b(lambda') -b(lambda))sum_T q^maj(T)
      endalign*



      Now, it is clear from the q-HLF formula that $q^maj(T)$ is a symmetric polynomial, with lowest degree term $b(lambda)$ and maximal degree $b(lambda) + binomn+12 - n -b(lambda) -b(lambda') =binomn2 - b(lambda')$ so the median degree term is
      $$M=frac12 left(b(lambda) +binomn2 - b(lambda')right)$$
      which cancels with the factor of $q$ in $f^lambda_F$, so the resulting polynomial is of the form
      beginalign*
      f^lambda_F = (-1)^M sum_T: maj(T) leq M (q^M-maj(T) + q^maj(T)-M) \
      = (-1)^M sum_T (-1)^M-maj(T)( varphi^2(M-maj(T)) + psi^2(M-maj(T)) =
      sum_T (-1)^maj(T) L(2(M-maj(T)))
      endalign*

      where $L$ are the Lucas numbers.



      **byproduct of collaborations with A. Morales and I. Pak.






      share|cite|improve this answer









      $endgroup$















        2












        2








        2





        $begingroup$

        This is a less elementary but maybe more conceptual proof, also giving some combinatorial meaning:
        Use the formulas
        $F(n) = fracvarphi^n -psi^nsqrt5$, $varphi =frac1+sqrt52, psi = frac1-sqrt52$. Let $q=fracpsivarphi = fracsqrt5-32$, so that
        $F(n) = fracvarphi^nsqrt5 (1-q^n)$



        Then the Fibonacci hook-length formula becomes:
        DeclareMathOperatorlalambda



        beginalign*
        f^lambda_F:= frac[n]!_Fprod_uin lambdaF(h(u)) = frac varphi^ binomn+12 [n]!_q varphi^sum_u in lambda h(u) prod_u in lambda (1-q^h(u))
        endalign*

        So we have an ordinary $q$-analogue of the hook-length formula. Note that
        $$sum_u in lambda h(u) = sum_i binomlambda_i2 + binomlambda'_j2 + |lambda| = b(lambda) +b(lambda') +n$$
        Using the $q-$analogue hook-length formula via major index (EC2, Chapter 21) we have



        beginalign*
        f^lambda_F = varphi^ binomn2 -b(lambda)-b(lambda') q^-b(lambda) sum_Tin SYT(lambda) q^maj(T) = (-q)^frac12( -binomn2 +b(lambda') -b(lambda))sum_T q^maj(T)
        endalign*



        Now, it is clear from the q-HLF formula that $q^maj(T)$ is a symmetric polynomial, with lowest degree term $b(lambda)$ and maximal degree $b(lambda) + binomn+12 - n -b(lambda) -b(lambda') =binomn2 - b(lambda')$ so the median degree term is
        $$M=frac12 left(b(lambda) +binomn2 - b(lambda')right)$$
        which cancels with the factor of $q$ in $f^lambda_F$, so the resulting polynomial is of the form
        beginalign*
        f^lambda_F = (-1)^M sum_T: maj(T) leq M (q^M-maj(T) + q^maj(T)-M) \
        = (-1)^M sum_T (-1)^M-maj(T)( varphi^2(M-maj(T)) + psi^2(M-maj(T)) =
        sum_T (-1)^maj(T) L(2(M-maj(T)))
        endalign*

        where $L$ are the Lucas numbers.



        **byproduct of collaborations with A. Morales and I. Pak.






        share|cite|improve this answer









        $endgroup$



        This is a less elementary but maybe more conceptual proof, also giving some combinatorial meaning:
        Use the formulas
        $F(n) = fracvarphi^n -psi^nsqrt5$, $varphi =frac1+sqrt52, psi = frac1-sqrt52$. Let $q=fracpsivarphi = fracsqrt5-32$, so that
        $F(n) = fracvarphi^nsqrt5 (1-q^n)$



        Then the Fibonacci hook-length formula becomes:
        DeclareMathOperatorlalambda



        beginalign*
        f^lambda_F:= frac[n]!_Fprod_uin lambdaF(h(u)) = frac varphi^ binomn+12 [n]!_q varphi^sum_u in lambda h(u) prod_u in lambda (1-q^h(u))
        endalign*

        So we have an ordinary $q$-analogue of the hook-length formula. Note that
        $$sum_u in lambda h(u) = sum_i binomlambda_i2 + binomlambda'_j2 + |lambda| = b(lambda) +b(lambda') +n$$
        Using the $q-$analogue hook-length formula via major index (EC2, Chapter 21) we have



        beginalign*
        f^lambda_F = varphi^ binomn2 -b(lambda)-b(lambda') q^-b(lambda) sum_Tin SYT(lambda) q^maj(T) = (-q)^frac12( -binomn2 +b(lambda') -b(lambda))sum_T q^maj(T)
        endalign*



        Now, it is clear from the q-HLF formula that $q^maj(T)$ is a symmetric polynomial, with lowest degree term $b(lambda)$ and maximal degree $b(lambda) + binomn+12 - n -b(lambda) -b(lambda') =binomn2 - b(lambda')$ so the median degree term is
        $$M=frac12 left(b(lambda) +binomn2 - b(lambda')right)$$
        which cancels with the factor of $q$ in $f^lambda_F$, so the resulting polynomial is of the form
        beginalign*
        f^lambda_F = (-1)^M sum_T: maj(T) leq M (q^M-maj(T) + q^maj(T)-M) \
        = (-1)^M sum_T (-1)^M-maj(T)( varphi^2(M-maj(T)) + psi^2(M-maj(T)) =
        sum_T (-1)^maj(T) L(2(M-maj(T)))
        endalign*

        where $L$ are the Lucas numbers.



        **byproduct of collaborations with A. Morales and I. Pak.







        share|cite|improve this answer












        share|cite|improve this answer



        share|cite|improve this answer










        answered 41 mins ago









        Greta PanovaGreta Panova

        15613




        15613



























            draft saved

            draft discarded
















































            Thanks for contributing an answer to MathOverflow!


            • Please be sure to answer the question. Provide details and share your research!

            But avoid


            • Asking for help, clarification, or responding to other answers.

            • Making statements based on opinion; back them up with references or personal experience.

            Use MathJax to format equations. MathJax reference.


            To learn more, see our tips on writing great answers.




            draft saved


            draft discarded














            StackExchange.ready(
            function ()
            StackExchange.openid.initPostLogin('.new-post-login', 'https%3a%2f%2fmathoverflow.net%2fquestions%2f326860%2fhook-length-formula-fibonaccized%23new-answer', 'question_page');

            );

            Post as a guest















            Required, but never shown





















































            Required, but never shown














            Required, but never shown












            Required, but never shown







            Required, but never shown

































            Required, but never shown














            Required, but never shown












            Required, but never shown







            Required, but never shown







            Popular posts from this blog

            Wikipedia:Vital articles Мазмуну Biography - Өмүр баян Philosophy and psychology - Философия жана психология Religion - Дин Social sciences - Коомдук илимдер Language and literature - Тил жана адабият Science - Илим Technology - Технология Arts and recreation - Искусство жана эс алуу History and geography - Тарых жана география Навигация менюсу

            Bruxelas-Capital Índice Historia | Composición | Situación lingüística | Clima | Cidades irmandadas | Notas | Véxase tamén | Menú de navegacióneO uso das linguas en Bruxelas e a situación do neerlandés"Rexión de Bruxelas Capital"o orixinalSitio da rexiónPáxina de Bruselas no sitio da Oficina de Promoción Turística de Valonia e BruxelasMapa Interactivo da Rexión de Bruxelas-CapitaleeWorldCat332144929079854441105155190212ID28008674080552-90000 0001 0666 3698n94104302ID540940339365017018237

            What should I write in an apology letter, since I have decided not to join a company after accepting an offer letterShould I keep looking after accepting a job offer?What should I do when I've been verbally told I would get an offer letter, but still haven't gotten one after 4 weeks?Do I accept an offer from a company that I am not likely to join?New job hasn't confirmed starting date and I want to give current employer as much notice as possibleHow should I address my manager in my resignation letter?HR delayed background verification, now jobless as resignedNo email communication after accepting a formal written offer. How should I phrase the call?What should I do if after receiving a verbal offer letter I am informed that my written job offer is put on hold due to some internal issues?Should I inform the current employer that I am about to resign within 1-2 weeks since I have signed the offer letter and waiting for visa?What company will do, if I send their offer letter to another company